Recent content by physics baws

  1. P

    [Biomedical engineering] Question about ECG signal electrodes

    (this is my another account, can't remember the password at the moment :D) Whoa, that looks awesome! Thank you very much for that! I'll have a closer look when I come to the office. In the meantime, I tried to make a small drawing (before I saw this post) to see if I can...
  2. P

    Total mechanical energy of a point mass in an elliptical orbit

    The second Newton's law tells us that the force vector has the same direction as the acceleration vector. Therefore, since the acceleration is normal to the direction of the velocity, as you say, it cannot have tangential acceleration, only centripetal. Therefore, the centripetal force is...
  3. P

    Total mechanical energy of a point mass in an elliptical orbit

    @mfb Well, they are not the same all the time, they are only the same in these particular points, in the case of an ellipse, that is. If they were not the same in those points, wouldn't that mean that there would be a tangential acceleration as well, and therefore a tangential force? We know...
  4. P

    Total mechanical energy of a point mass in an elliptical orbit

    Hm, if you're referring to the speed of the other body, I just assumed it's zero. As in, if I am looking at the motion of the satellite orbiting the Sun, I assume that the Sun is stationary in its position. Edit: Oh, and if you are talking about my equations above, then \frac{mv_1...
  5. P

    Total mechanical energy of a point mass in an elliptical orbit

    Hi, Thank you for your response, and I am aware of google and wikipedia, however I am trying to find a mistake in my reasoning above.
  6. P

    Total mechanical energy of a point mass in an elliptical orbit

    Hey, I was trying to prove to myself an expression for the total energy, and I got stuck Here is the picture (it's transparent, so I won't embed it here). The problem I seem to be having is when I observe these two points where the speed is in its highest and its lowest. Since in these...
  7. P

    Variance of White Noise: How Can It Have Infinite Power?

    Sorry, I was away these few days. Yeah, that makes sense completely, thanks again :)
  8. P

    Variance of White Noise: How Can It Have Infinite Power?

    Ah, I see. That makes sense. Could you perhaps elaborate just a little bit more on as to why it has to be infinitely long? I'm not sure I see it. But other than that, I am all good! Thanks!
  9. P

    Variance of White Noise: How Can It Have Infinite Power?

    Hey, thanks for the answer. Let me try to make it clearer R_{x}(\tau) = E[X(t)X(t+\tau)] R_{x}(0) = E[X^{2}(t)] = \sigma^2 or R_{x}(\tau) = \int S(f) e^{j2\pi ft}df R_{x}(0) = \int S(f) df The white noise is defined by having a flat power spectral density over the whole range of...
  10. P

    Variance of White Noise: How Can It Have Infinite Power?

    Hi, I have a pretty simple question which I thought I do not need to make a topic about, but Google is actually not helping, which is surprising. So here it goes: How can white noise have infinite power if its variance is finite? As far as I am aware, the following is always valid for a...
  11. P

    The Zeno's paradox and one of its variations

    Hi, I was learning about collisions, and I stumbled upon this materials, which is interesting because a guy who wrote it gave this interesting example. He was talking about "somewhat inelastic collisions", as he calls them, and he gave an example of a ball bouncing of the floor. Here are the...
  12. P

    Statistics of random processes passed through an LTI system

    Hello, I apologize in advance if I have missed the right place to ask. I'd be grateful if you could forward me to the right place, if that is the case. Google didn't help, so maybe someone here can point me in the right direction: 1) "If the input to a LTI system is a Gaussian random...
  13. P

    Violation of energy conservation?

    Hm, wow, it seems I was living in a lie the whole time. I thought that \Delta K = -\Delta U was always true, no mater what forces are there, only the net force is considered. Thank you guys, it seems I got some thinking to do!
  14. P

    Violation of energy conservation?

    Hello, I have a question, which is kinda stupid probably, but I cannot solve it for myself. Hence the fancy title. Imagine you have a point mass m near a more massive object M. Let's say there is some other force supporting the point mass, so it can stand still at r = \vec{r_0}. Point mass...
Back
Top